0 of 20 Questions completed
Questions:
You have already completed the quiz before. Hence you can not start it again.
Quiz is loading…
You must sign in or sign up to start the quiz.
You must first complete the following:
0 of 20 Questions answered correctly
Your time:
Time has elapsed
You have reached 0 of 0 point(s), (0)
Earned Point(s): 0 of 0, (0)
0 Essay(s) Pending (Possible Point(s): 0)
Average score |
|
Your score |
|
An 11-month-old male is brought to the emergency room with agitation, fever, and nuchal rigidity. A review of the patient’s electronic medical record shows that the patient was admitted with meningitis 2 months ago. While no clear focal neurological symptoms were appreciated on examination, there was a midline dimple appreciated in the lumbosacral region. Which of the following is the most likely etiology of the patient’s recurrent illness?
The brain MRI (susceptibility-weighted imaging) shown below is most likely associated with which of the following conditions?
A 68-year-old right-handed male with HTN and atrial fibrillation formerly taking Eliquis, but stopped months ago due to GI bleed, presents for acute onset neurologic symptoms. He is able to understand speech and commands, however, he is unable to speak fluently and can not get the right words out. Despite this, he is able to accurately repeat words you say and correctly name items that are pointed to. What other neurologic symptom is also likely to be present on the exam?
A 65-year-old female presents with complaints of hearing a strange “clicking sound” all of the time which started suddenly 6 weeks ago. On exam, you note a rhythmic tremor of the upper palate. The patient also has pendular horizontal nystagmus. Which MRI finding is most likely?
Which of the following areas, if selectively lesioned, would cause an error with voluntary vertical saccades with spared horizontal saccades and sparred occulocephalic vertical reflex?
You are interested in initiating tetrabenazine for a patient with refractory chorea. You should educate the patient that this medication carries a black box warning for which of the following?
A patient with a history of hypertension and provoked seizures secondary to posterior reversible encephalopathy syndrome (PRES) has come to your clinic for a 6-month follow-up after her associated hospitalization. It was found that abrupt discontinuation of her hypertensive medications due to medication noncompliance was the cause of the PRES. She was started on the anti-seizure medication (ASM) levetiracetam at the time of her PRES and has had no seizures since being discharged. She asks you about the risk of future seizures. What is the most appropriate response?
A 64-year-old man with idiopathic Parkinson’s disease being treated with carbidopa-levodopa is experiencing significant wearing-off despite increasing frequency and dose of levodopa-carbidopa, which is now producing significant peak dose dyskinesia. Which of the following would be a reasonable next agent to prescribe to reduce this wearing-off effect?
Which of the following vessels that supply the thalamus typically originates from the posterior communicating artery (PCOM)?
A 35-year-old male presented for sudden onset painless central vision loss. An outside hospital emergency room physician suggested he may have had a CRAO, but recommended immediate ophthalmology outpatient evaluation. The patient was unfortunately lost to follow-up for a few months. He presents today due to a sudden onset of central vision loss in the other eye.
On your ophthalmologic exam, you note hyperemic optic nerve and tortuous central retinal vessels, without evidence of vessel cutoff. He has a central scotoma bilaterally, with peripheral vision intact. He has red color desaturation as well. MRI brain w/ and w/o contrast with fat suppression and thin cuts through the orbits is performed, and normal.
This disorder is caused by which of the following pathologic processes?
Which of the following is the most likely cause of the findings seen on this gross pathology image?
Which of the following cranial nerves is most likely to be affected by increased intracranial pressure?
A 34-year-old man was admitted to the hospital and intubated after a severe motor vehicle accident leading to multiple bone fractures two days ago. Despite weaning IV sedation, the patient continues to be unarousable to stimulation. An MRI was performed and DWI imaging is shown below. What is the most likely diagnosis?
An MRI is ordered on a man with a known history of ischemic stroke and bilateral upper extremity weakness. Which of the following was the most likely cause of his ischemic injury?
A 30-year-old man with a past medical history of migraines presents to the emergency department for his typical migraine symptoms, which have become intractable. He is treated with a “migraine cocktail” (normal saline, acetaminophen, and prochlorperazine) with some improvement. You are then called to the bedside for concern that he is having a possible seizure. His head is turned forcibly to the right without eye deviation and he is able to follow commands and verbalize. Which of the following is the best next step in management?
A 51-year-old woman with a past medical history of migraines and irritable bowel syndrome comes to the physician because of abnormal shaking of both hands for the past two months. The shaking improves after two glasses of wine. Examination shows symmetric hand tremors that worsen when the patient reaches for objects in front of her. What is the mechanism of action of the most appropriate first-line treatment?
Deep brain stimulation has been effective in the management of Parkinson’s disease when it targets which of the following structures?
Which of the following nuclei of the hypothalamus is involved in temperature regulation?
A 63-year-old female presents to the office, accompanied by her husband who has become increasingly worried about his wife’s ability to perform tasks at home. He states that for the past three months she has not been able to tie her shoes, and fumbles with the shoelaces in her hands. Additionally, within the past month, she has not been able to brush her teeth, and simply holds the toothbrush in her hand, without moving. Lastly, her husband has noticed that the patient moves slower on their morning walks together, and her arms and legs seem more tight and rigid. A brain MRI is ordered which shows asymmetric atrophy of the superior parietal lobule. Which of the following is the most likely diagnosis?
What is the mechanism of action of apixaban?